Why is $x+e^{-x}>0$ for all $x in mathbb{R}$? The 2019 Stack Overflow Developer Survey...

I looked up a future colleague on LinkedIn before I started a job. I told my colleague about it and he seemed surprised. Should I apologize?

What is the motivation for a law requiring 2 parties to consent for recording a conversation

How long do I have to send payment?

If a poisoned arrow's piercing damage is reduced to 0, do you still get poisoned?

Why isn't airport relocation done gradually?

Landlord wants to switch my lease to a "Land contract" to "get back at the city"

Carnot-Caratheodory metric

Are there any other methods to apply to solving simultaneous equations?

Where does the "burst of radiance" from Holy Weapon originate?

How to reverse every other sublist of a list?

Spanish for "widget"

What does "sndry explns" mean in one of the Hitchhiker's guide books?

Should I write numbers in words or as numerals when there are multiple next to each other?

What does "rabbited" mean/imply in this sentence?

Springs with some finite mass

Realistic Alternatives to Dust: What Else Could Feed a Plankton Bloom?

Where to refill my bottle in India?

Pristine Bit Checking

What is the use of option -o in the useradd command?

Could JWST stay at L2 "forever"?

Why is the maximum length of OpenWrt’s root password 8 characters?

The difference between dialogue marks

Output the Arecibo Message

aging parents with no investments



Why is $x+e^{-x}>0$ for all $x in mathbb{R}$?



The 2019 Stack Overflow Developer Survey Results Are InSimplest or nicest proof that $1+x le e^x$What is the limit of this function as $n$ tends to infinity? (something to the power of e)Limit question: $lim_{xto infty}f(x),;; lim_{xto -infty}f(x)$?Does $displaystylelim_{ntoinfty}frac{n}{n+1} = 1$?Calculus - Finding the rate of change of an edge of a cube.Calculus -as X approaches O or infinityProve limit exists for an almost monotonic bounded function, involves convexity and square integrabilityCan I not use L'hopital rule here?What do second order partial derivatives mean in graphs?Find the simplest counterexample against exchanging limit and summationIf $f : [a, infty) → Bbb R$ is monotonically decreasing and the integral $int_0^infty f(x) ,dx$ is convergent, then $lim_{x→infty} f(x) = 0$.












2












$begingroup$


Denote $f(x) = x + e^{-x}$. Note that $f(0) = 1$ and $f'(x) = 1-e^{-x}$. That means
$$lim_{xrightarrow -infty} f'(x) = -infty.$$
So if the rate of change of $f(x)$ keeps decreasing exponentially fast to negative infinity, why is $lim_{xrightarrow -infty} f(x) = +infty$?



I think I am missing something really simple here. Please help.










share|cite|improve this question











$endgroup$












  • $begingroup$
    Addressing the title, for all $x, e^{-x}ge1-x$ so $x+e^{-x}ge1>0$
    $endgroup$
    – J. W. Tanner
    Mar 21 at 4:45








  • 1




    $begingroup$
    To show $x+e^{-x}> 0$, you want to show $e^{-x}> -x$. In fact, we have $e^{t}ge t+1$ for all real $t$. So $e^{-x}ge -x+1> -x$ for all real $x$, as desired.
    $endgroup$
    – Minus One-Twelfth
    Mar 21 at 4:46








  • 4




    $begingroup$
    Roughly speaking - if you are visualising $x$ going from $0$ to $-infty$, then the graph is not decreasing but increasing exponentially fast, because you are going "backwards".
    $endgroup$
    – David
    Mar 21 at 4:49








  • 1




    $begingroup$
    Thank you. That's what I was missing. Going backward. :(
    $endgroup$
    – Paichu
    Mar 21 at 4:50
















2












$begingroup$


Denote $f(x) = x + e^{-x}$. Note that $f(0) = 1$ and $f'(x) = 1-e^{-x}$. That means
$$lim_{xrightarrow -infty} f'(x) = -infty.$$
So if the rate of change of $f(x)$ keeps decreasing exponentially fast to negative infinity, why is $lim_{xrightarrow -infty} f(x) = +infty$?



I think I am missing something really simple here. Please help.










share|cite|improve this question











$endgroup$












  • $begingroup$
    Addressing the title, for all $x, e^{-x}ge1-x$ so $x+e^{-x}ge1>0$
    $endgroup$
    – J. W. Tanner
    Mar 21 at 4:45








  • 1




    $begingroup$
    To show $x+e^{-x}> 0$, you want to show $e^{-x}> -x$. In fact, we have $e^{t}ge t+1$ for all real $t$. So $e^{-x}ge -x+1> -x$ for all real $x$, as desired.
    $endgroup$
    – Minus One-Twelfth
    Mar 21 at 4:46








  • 4




    $begingroup$
    Roughly speaking - if you are visualising $x$ going from $0$ to $-infty$, then the graph is not decreasing but increasing exponentially fast, because you are going "backwards".
    $endgroup$
    – David
    Mar 21 at 4:49








  • 1




    $begingroup$
    Thank you. That's what I was missing. Going backward. :(
    $endgroup$
    – Paichu
    Mar 21 at 4:50














2












2








2


2



$begingroup$


Denote $f(x) = x + e^{-x}$. Note that $f(0) = 1$ and $f'(x) = 1-e^{-x}$. That means
$$lim_{xrightarrow -infty} f'(x) = -infty.$$
So if the rate of change of $f(x)$ keeps decreasing exponentially fast to negative infinity, why is $lim_{xrightarrow -infty} f(x) = +infty$?



I think I am missing something really simple here. Please help.










share|cite|improve this question











$endgroup$




Denote $f(x) = x + e^{-x}$. Note that $f(0) = 1$ and $f'(x) = 1-e^{-x}$. That means
$$lim_{xrightarrow -infty} f'(x) = -infty.$$
So if the rate of change of $f(x)$ keeps decreasing exponentially fast to negative infinity, why is $lim_{xrightarrow -infty} f(x) = +infty$?



I think I am missing something really simple here. Please help.







calculus limits






share|cite|improve this question















share|cite|improve this question













share|cite|improve this question




share|cite|improve this question








edited Mar 21 at 6:31









David G. Stork

12.2k41836




12.2k41836










asked Mar 21 at 4:42









PaichuPaichu

804616




804616












  • $begingroup$
    Addressing the title, for all $x, e^{-x}ge1-x$ so $x+e^{-x}ge1>0$
    $endgroup$
    – J. W. Tanner
    Mar 21 at 4:45








  • 1




    $begingroup$
    To show $x+e^{-x}> 0$, you want to show $e^{-x}> -x$. In fact, we have $e^{t}ge t+1$ for all real $t$. So $e^{-x}ge -x+1> -x$ for all real $x$, as desired.
    $endgroup$
    – Minus One-Twelfth
    Mar 21 at 4:46








  • 4




    $begingroup$
    Roughly speaking - if you are visualising $x$ going from $0$ to $-infty$, then the graph is not decreasing but increasing exponentially fast, because you are going "backwards".
    $endgroup$
    – David
    Mar 21 at 4:49








  • 1




    $begingroup$
    Thank you. That's what I was missing. Going backward. :(
    $endgroup$
    – Paichu
    Mar 21 at 4:50


















  • $begingroup$
    Addressing the title, for all $x, e^{-x}ge1-x$ so $x+e^{-x}ge1>0$
    $endgroup$
    – J. W. Tanner
    Mar 21 at 4:45








  • 1




    $begingroup$
    To show $x+e^{-x}> 0$, you want to show $e^{-x}> -x$. In fact, we have $e^{t}ge t+1$ for all real $t$. So $e^{-x}ge -x+1> -x$ for all real $x$, as desired.
    $endgroup$
    – Minus One-Twelfth
    Mar 21 at 4:46








  • 4




    $begingroup$
    Roughly speaking - if you are visualising $x$ going from $0$ to $-infty$, then the graph is not decreasing but increasing exponentially fast, because you are going "backwards".
    $endgroup$
    – David
    Mar 21 at 4:49








  • 1




    $begingroup$
    Thank you. That's what I was missing. Going backward. :(
    $endgroup$
    – Paichu
    Mar 21 at 4:50
















$begingroup$
Addressing the title, for all $x, e^{-x}ge1-x$ so $x+e^{-x}ge1>0$
$endgroup$
– J. W. Tanner
Mar 21 at 4:45






$begingroup$
Addressing the title, for all $x, e^{-x}ge1-x$ so $x+e^{-x}ge1>0$
$endgroup$
– J. W. Tanner
Mar 21 at 4:45






1




1




$begingroup$
To show $x+e^{-x}> 0$, you want to show $e^{-x}> -x$. In fact, we have $e^{t}ge t+1$ for all real $t$. So $e^{-x}ge -x+1> -x$ for all real $x$, as desired.
$endgroup$
– Minus One-Twelfth
Mar 21 at 4:46






$begingroup$
To show $x+e^{-x}> 0$, you want to show $e^{-x}> -x$. In fact, we have $e^{t}ge t+1$ for all real $t$. So $e^{-x}ge -x+1> -x$ for all real $x$, as desired.
$endgroup$
– Minus One-Twelfth
Mar 21 at 4:46






4




4




$begingroup$
Roughly speaking - if you are visualising $x$ going from $0$ to $-infty$, then the graph is not decreasing but increasing exponentially fast, because you are going "backwards".
$endgroup$
– David
Mar 21 at 4:49






$begingroup$
Roughly speaking - if you are visualising $x$ going from $0$ to $-infty$, then the graph is not decreasing but increasing exponentially fast, because you are going "backwards".
$endgroup$
– David
Mar 21 at 4:49






1




1




$begingroup$
Thank you. That's what I was missing. Going backward. :(
$endgroup$
– Paichu
Mar 21 at 4:50




$begingroup$
Thank you. That's what I was missing. Going backward. :(
$endgroup$
– Paichu
Mar 21 at 4:50










4 Answers
4






active

oldest

votes


















2












$begingroup$

You have got your directions mixed up.



When the derivative is negative, the function is decreasing from left to right i.e. its graph is coming down from left to right. Which is the same as , going right to left its graph is going up or it is increasing, which is the phenomena observed here.






share|cite|improve this answer









$endgroup$





















    3












    $begingroup$

    Take



    $x < y < 0; tag 1$



    then



    $f(y) - f(x) = displaystyle int_x^y f'(s); ds = int_x^y (1 - e^{-s}); ds; tag 2$



    it is clear that, for any $M < 0$, there exists $y_0 < 0$ such that



    $s < y_0 Longrightarrow 1 - e^{-s} < M; tag 3$



    if we now choose



    $y < y_0, tag 4$



    then



    $f(y) - f(x) = displaystyle int_x^y (1 - e^{-s}); ds < int_x^y M ; ds = M(y - x); tag 5$



    we re-arrange this inequality:



    $f(x) - f(y) > -M(y - x) = M(x - y), tag 6$



    $f(x) > f(y) + M(x - y); tag 7$



    we now fix $y$ and let $x to -infty$; then since $M < 0$ and $x - y < 0$ for $x < y$,



    $displaystyle lim_{x to -infty} f(y) + M(x - y) = infty, tag 8$



    and hence



    $displaystyle lim_{x to -infty}f(x) = infty tag 9$



    as well.



    We note that (9) binds despite the fact that $f'(x) < 0$ for $x < 0$; though this derivative is negative, when $x$ decreases we are "walking back up the hill," as it were; as $x$ decreases, $f(x)$ increases.



    Note Added in Edit, Wednesday 20 March 2019 10:39 PM PST: We may also dispense with the title question and show



    $x + e^{-x} > 0, forall x in Bbb R; tag{10}$



    for



    $x ge 0, tag{11}$



    $e^{-x} > 0 tag{12}$



    as well, hence we also have



    $x + e^{-x} > 0; tag{13}$



    for



    $x < 0, tag{14}$



    we may use the power series for $e^{-x}$:



    $e^{-x} = 1 - x + dfrac{x^2}{2!} - dfrac{x^3}{3} + ldots; tag{15}$



    then



    $x + e^{-x} = 1 + dfrac{x^2}{2!} - dfrac{x^3}{3!} + ldots > 0, tag{16}$



    since every term on the right is positive when $x < 0$. End of Note.






    share|cite|improve this answer











    $endgroup$





















      2












      $begingroup$

      Note that $f''(x) = e^{-x} >0$ so $f$ is strictly convex.



      Since $f'(0) = 0$, we see that $f(x) ge f(0) = 1$ for all $x$.






      share|cite|improve this answer









      $endgroup$





















        0












        $begingroup$

        Note that




        • $forall x in mathbb{R} : x+e^{-x}>0 Leftrightarrow forall x in mathbb{R} :e^{-x}>-x Leftrightarrow forall x in mathbb{R} :color{blue}{boxed{e^{x}>x}}$


        The last inequality follows directly by Taylor:
        $$color{blue}{e^x} = 1+x + underbrace{frac{e^{xi}}{2}x^2}_{geq 0} color{blue}{> x}$$






        share|cite|improve this answer









        $endgroup$














          Your Answer





          StackExchange.ifUsing("editor", function () {
          return StackExchange.using("mathjaxEditing", function () {
          StackExchange.MarkdownEditor.creationCallbacks.add(function (editor, postfix) {
          StackExchange.mathjaxEditing.prepareWmdForMathJax(editor, postfix, [["$", "$"], ["\\(","\\)"]]);
          });
          });
          }, "mathjax-editing");

          StackExchange.ready(function() {
          var channelOptions = {
          tags: "".split(" "),
          id: "69"
          };
          initTagRenderer("".split(" "), "".split(" "), channelOptions);

          StackExchange.using("externalEditor", function() {
          // Have to fire editor after snippets, if snippets enabled
          if (StackExchange.settings.snippets.snippetsEnabled) {
          StackExchange.using("snippets", function() {
          createEditor();
          });
          }
          else {
          createEditor();
          }
          });

          function createEditor() {
          StackExchange.prepareEditor({
          heartbeatType: 'answer',
          autoActivateHeartbeat: false,
          convertImagesToLinks: true,
          noModals: true,
          showLowRepImageUploadWarning: true,
          reputationToPostImages: 10,
          bindNavPrevention: true,
          postfix: "",
          imageUploader: {
          brandingHtml: "Powered by u003ca class="icon-imgur-white" href="https://imgur.com/"u003eu003c/au003e",
          contentPolicyHtml: "User contributions licensed under u003ca href="https://creativecommons.org/licenses/by-sa/3.0/"u003ecc by-sa 3.0 with attribution requiredu003c/au003e u003ca href="https://stackoverflow.com/legal/content-policy"u003e(content policy)u003c/au003e",
          allowUrls: true
          },
          noCode: true, onDemand: true,
          discardSelector: ".discard-answer"
          ,immediatelyShowMarkdownHelp:true
          });


          }
          });














          draft saved

          draft discarded


















          StackExchange.ready(
          function () {
          StackExchange.openid.initPostLogin('.new-post-login', 'https%3a%2f%2fmath.stackexchange.com%2fquestions%2f3156341%2fwhy-is-xe-x0-for-all-x-in-mathbbr%23new-answer', 'question_page');
          }
          );

          Post as a guest















          Required, but never shown

























          4 Answers
          4






          active

          oldest

          votes








          4 Answers
          4






          active

          oldest

          votes









          active

          oldest

          votes






          active

          oldest

          votes









          2












          $begingroup$

          You have got your directions mixed up.



          When the derivative is negative, the function is decreasing from left to right i.e. its graph is coming down from left to right. Which is the same as , going right to left its graph is going up or it is increasing, which is the phenomena observed here.






          share|cite|improve this answer









          $endgroup$


















            2












            $begingroup$

            You have got your directions mixed up.



            When the derivative is negative, the function is decreasing from left to right i.e. its graph is coming down from left to right. Which is the same as , going right to left its graph is going up or it is increasing, which is the phenomena observed here.






            share|cite|improve this answer









            $endgroup$
















              2












              2








              2





              $begingroup$

              You have got your directions mixed up.



              When the derivative is negative, the function is decreasing from left to right i.e. its graph is coming down from left to right. Which is the same as , going right to left its graph is going up or it is increasing, which is the phenomena observed here.






              share|cite|improve this answer









              $endgroup$



              You have got your directions mixed up.



              When the derivative is negative, the function is decreasing from left to right i.e. its graph is coming down from left to right. Which is the same as , going right to left its graph is going up or it is increasing, which is the phenomena observed here.







              share|cite|improve this answer












              share|cite|improve this answer



              share|cite|improve this answer










              answered Mar 21 at 4:51









              астон вілла олоф мэллбэргастон вілла олоф мэллбэрг

              40.3k33678




              40.3k33678























                  3












                  $begingroup$

                  Take



                  $x < y < 0; tag 1$



                  then



                  $f(y) - f(x) = displaystyle int_x^y f'(s); ds = int_x^y (1 - e^{-s}); ds; tag 2$



                  it is clear that, for any $M < 0$, there exists $y_0 < 0$ such that



                  $s < y_0 Longrightarrow 1 - e^{-s} < M; tag 3$



                  if we now choose



                  $y < y_0, tag 4$



                  then



                  $f(y) - f(x) = displaystyle int_x^y (1 - e^{-s}); ds < int_x^y M ; ds = M(y - x); tag 5$



                  we re-arrange this inequality:



                  $f(x) - f(y) > -M(y - x) = M(x - y), tag 6$



                  $f(x) > f(y) + M(x - y); tag 7$



                  we now fix $y$ and let $x to -infty$; then since $M < 0$ and $x - y < 0$ for $x < y$,



                  $displaystyle lim_{x to -infty} f(y) + M(x - y) = infty, tag 8$



                  and hence



                  $displaystyle lim_{x to -infty}f(x) = infty tag 9$



                  as well.



                  We note that (9) binds despite the fact that $f'(x) < 0$ for $x < 0$; though this derivative is negative, when $x$ decreases we are "walking back up the hill," as it were; as $x$ decreases, $f(x)$ increases.



                  Note Added in Edit, Wednesday 20 March 2019 10:39 PM PST: We may also dispense with the title question and show



                  $x + e^{-x} > 0, forall x in Bbb R; tag{10}$



                  for



                  $x ge 0, tag{11}$



                  $e^{-x} > 0 tag{12}$



                  as well, hence we also have



                  $x + e^{-x} > 0; tag{13}$



                  for



                  $x < 0, tag{14}$



                  we may use the power series for $e^{-x}$:



                  $e^{-x} = 1 - x + dfrac{x^2}{2!} - dfrac{x^3}{3} + ldots; tag{15}$



                  then



                  $x + e^{-x} = 1 + dfrac{x^2}{2!} - dfrac{x^3}{3!} + ldots > 0, tag{16}$



                  since every term on the right is positive when $x < 0$. End of Note.






                  share|cite|improve this answer











                  $endgroup$


















                    3












                    $begingroup$

                    Take



                    $x < y < 0; tag 1$



                    then



                    $f(y) - f(x) = displaystyle int_x^y f'(s); ds = int_x^y (1 - e^{-s}); ds; tag 2$



                    it is clear that, for any $M < 0$, there exists $y_0 < 0$ such that



                    $s < y_0 Longrightarrow 1 - e^{-s} < M; tag 3$



                    if we now choose



                    $y < y_0, tag 4$



                    then



                    $f(y) - f(x) = displaystyle int_x^y (1 - e^{-s}); ds < int_x^y M ; ds = M(y - x); tag 5$



                    we re-arrange this inequality:



                    $f(x) - f(y) > -M(y - x) = M(x - y), tag 6$



                    $f(x) > f(y) + M(x - y); tag 7$



                    we now fix $y$ and let $x to -infty$; then since $M < 0$ and $x - y < 0$ for $x < y$,



                    $displaystyle lim_{x to -infty} f(y) + M(x - y) = infty, tag 8$



                    and hence



                    $displaystyle lim_{x to -infty}f(x) = infty tag 9$



                    as well.



                    We note that (9) binds despite the fact that $f'(x) < 0$ for $x < 0$; though this derivative is negative, when $x$ decreases we are "walking back up the hill," as it were; as $x$ decreases, $f(x)$ increases.



                    Note Added in Edit, Wednesday 20 March 2019 10:39 PM PST: We may also dispense with the title question and show



                    $x + e^{-x} > 0, forall x in Bbb R; tag{10}$



                    for



                    $x ge 0, tag{11}$



                    $e^{-x} > 0 tag{12}$



                    as well, hence we also have



                    $x + e^{-x} > 0; tag{13}$



                    for



                    $x < 0, tag{14}$



                    we may use the power series for $e^{-x}$:



                    $e^{-x} = 1 - x + dfrac{x^2}{2!} - dfrac{x^3}{3} + ldots; tag{15}$



                    then



                    $x + e^{-x} = 1 + dfrac{x^2}{2!} - dfrac{x^3}{3!} + ldots > 0, tag{16}$



                    since every term on the right is positive when $x < 0$. End of Note.






                    share|cite|improve this answer











                    $endgroup$
















                      3












                      3








                      3





                      $begingroup$

                      Take



                      $x < y < 0; tag 1$



                      then



                      $f(y) - f(x) = displaystyle int_x^y f'(s); ds = int_x^y (1 - e^{-s}); ds; tag 2$



                      it is clear that, for any $M < 0$, there exists $y_0 < 0$ such that



                      $s < y_0 Longrightarrow 1 - e^{-s} < M; tag 3$



                      if we now choose



                      $y < y_0, tag 4$



                      then



                      $f(y) - f(x) = displaystyle int_x^y (1 - e^{-s}); ds < int_x^y M ; ds = M(y - x); tag 5$



                      we re-arrange this inequality:



                      $f(x) - f(y) > -M(y - x) = M(x - y), tag 6$



                      $f(x) > f(y) + M(x - y); tag 7$



                      we now fix $y$ and let $x to -infty$; then since $M < 0$ and $x - y < 0$ for $x < y$,



                      $displaystyle lim_{x to -infty} f(y) + M(x - y) = infty, tag 8$



                      and hence



                      $displaystyle lim_{x to -infty}f(x) = infty tag 9$



                      as well.



                      We note that (9) binds despite the fact that $f'(x) < 0$ for $x < 0$; though this derivative is negative, when $x$ decreases we are "walking back up the hill," as it were; as $x$ decreases, $f(x)$ increases.



                      Note Added in Edit, Wednesday 20 March 2019 10:39 PM PST: We may also dispense with the title question and show



                      $x + e^{-x} > 0, forall x in Bbb R; tag{10}$



                      for



                      $x ge 0, tag{11}$



                      $e^{-x} > 0 tag{12}$



                      as well, hence we also have



                      $x + e^{-x} > 0; tag{13}$



                      for



                      $x < 0, tag{14}$



                      we may use the power series for $e^{-x}$:



                      $e^{-x} = 1 - x + dfrac{x^2}{2!} - dfrac{x^3}{3} + ldots; tag{15}$



                      then



                      $x + e^{-x} = 1 + dfrac{x^2}{2!} - dfrac{x^3}{3!} + ldots > 0, tag{16}$



                      since every term on the right is positive when $x < 0$. End of Note.






                      share|cite|improve this answer











                      $endgroup$



                      Take



                      $x < y < 0; tag 1$



                      then



                      $f(y) - f(x) = displaystyle int_x^y f'(s); ds = int_x^y (1 - e^{-s}); ds; tag 2$



                      it is clear that, for any $M < 0$, there exists $y_0 < 0$ such that



                      $s < y_0 Longrightarrow 1 - e^{-s} < M; tag 3$



                      if we now choose



                      $y < y_0, tag 4$



                      then



                      $f(y) - f(x) = displaystyle int_x^y (1 - e^{-s}); ds < int_x^y M ; ds = M(y - x); tag 5$



                      we re-arrange this inequality:



                      $f(x) - f(y) > -M(y - x) = M(x - y), tag 6$



                      $f(x) > f(y) + M(x - y); tag 7$



                      we now fix $y$ and let $x to -infty$; then since $M < 0$ and $x - y < 0$ for $x < y$,



                      $displaystyle lim_{x to -infty} f(y) + M(x - y) = infty, tag 8$



                      and hence



                      $displaystyle lim_{x to -infty}f(x) = infty tag 9$



                      as well.



                      We note that (9) binds despite the fact that $f'(x) < 0$ for $x < 0$; though this derivative is negative, when $x$ decreases we are "walking back up the hill," as it were; as $x$ decreases, $f(x)$ increases.



                      Note Added in Edit, Wednesday 20 March 2019 10:39 PM PST: We may also dispense with the title question and show



                      $x + e^{-x} > 0, forall x in Bbb R; tag{10}$



                      for



                      $x ge 0, tag{11}$



                      $e^{-x} > 0 tag{12}$



                      as well, hence we also have



                      $x + e^{-x} > 0; tag{13}$



                      for



                      $x < 0, tag{14}$



                      we may use the power series for $e^{-x}$:



                      $e^{-x} = 1 - x + dfrac{x^2}{2!} - dfrac{x^3}{3} + ldots; tag{15}$



                      then



                      $x + e^{-x} = 1 + dfrac{x^2}{2!} - dfrac{x^3}{3!} + ldots > 0, tag{16}$



                      since every term on the right is positive when $x < 0$. End of Note.







                      share|cite|improve this answer














                      share|cite|improve this answer



                      share|cite|improve this answer








                      edited Mar 21 at 5:46

























                      answered Mar 21 at 5:29









                      Robert LewisRobert Lewis

                      48.9k23168




                      48.9k23168























                          2












                          $begingroup$

                          Note that $f''(x) = e^{-x} >0$ so $f$ is strictly convex.



                          Since $f'(0) = 0$, we see that $f(x) ge f(0) = 1$ for all $x$.






                          share|cite|improve this answer









                          $endgroup$


















                            2












                            $begingroup$

                            Note that $f''(x) = e^{-x} >0$ so $f$ is strictly convex.



                            Since $f'(0) = 0$, we see that $f(x) ge f(0) = 1$ for all $x$.






                            share|cite|improve this answer









                            $endgroup$
















                              2












                              2








                              2





                              $begingroup$

                              Note that $f''(x) = e^{-x} >0$ so $f$ is strictly convex.



                              Since $f'(0) = 0$, we see that $f(x) ge f(0) = 1$ for all $x$.






                              share|cite|improve this answer









                              $endgroup$



                              Note that $f''(x) = e^{-x} >0$ so $f$ is strictly convex.



                              Since $f'(0) = 0$, we see that $f(x) ge f(0) = 1$ for all $x$.







                              share|cite|improve this answer












                              share|cite|improve this answer



                              share|cite|improve this answer










                              answered Mar 21 at 4:51









                              copper.hatcopper.hat

                              128k561161




                              128k561161























                                  0












                                  $begingroup$

                                  Note that




                                  • $forall x in mathbb{R} : x+e^{-x}>0 Leftrightarrow forall x in mathbb{R} :e^{-x}>-x Leftrightarrow forall x in mathbb{R} :color{blue}{boxed{e^{x}>x}}$


                                  The last inequality follows directly by Taylor:
                                  $$color{blue}{e^x} = 1+x + underbrace{frac{e^{xi}}{2}x^2}_{geq 0} color{blue}{> x}$$






                                  share|cite|improve this answer









                                  $endgroup$


















                                    0












                                    $begingroup$

                                    Note that




                                    • $forall x in mathbb{R} : x+e^{-x}>0 Leftrightarrow forall x in mathbb{R} :e^{-x}>-x Leftrightarrow forall x in mathbb{R} :color{blue}{boxed{e^{x}>x}}$


                                    The last inequality follows directly by Taylor:
                                    $$color{blue}{e^x} = 1+x + underbrace{frac{e^{xi}}{2}x^2}_{geq 0} color{blue}{> x}$$






                                    share|cite|improve this answer









                                    $endgroup$
















                                      0












                                      0








                                      0





                                      $begingroup$

                                      Note that




                                      • $forall x in mathbb{R} : x+e^{-x}>0 Leftrightarrow forall x in mathbb{R} :e^{-x}>-x Leftrightarrow forall x in mathbb{R} :color{blue}{boxed{e^{x}>x}}$


                                      The last inequality follows directly by Taylor:
                                      $$color{blue}{e^x} = 1+x + underbrace{frac{e^{xi}}{2}x^2}_{geq 0} color{blue}{> x}$$






                                      share|cite|improve this answer









                                      $endgroup$



                                      Note that




                                      • $forall x in mathbb{R} : x+e^{-x}>0 Leftrightarrow forall x in mathbb{R} :e^{-x}>-x Leftrightarrow forall x in mathbb{R} :color{blue}{boxed{e^{x}>x}}$


                                      The last inequality follows directly by Taylor:
                                      $$color{blue}{e^x} = 1+x + underbrace{frac{e^{xi}}{2}x^2}_{geq 0} color{blue}{> x}$$







                                      share|cite|improve this answer












                                      share|cite|improve this answer



                                      share|cite|improve this answer










                                      answered Mar 21 at 7:03









                                      trancelocationtrancelocation

                                      13.7k1829




                                      13.7k1829






























                                          draft saved

                                          draft discarded




















































                                          Thanks for contributing an answer to Mathematics Stack Exchange!


                                          • Please be sure to answer the question. Provide details and share your research!

                                          But avoid



                                          • Asking for help, clarification, or responding to other answers.

                                          • Making statements based on opinion; back them up with references or personal experience.


                                          Use MathJax to format equations. MathJax reference.


                                          To learn more, see our tips on writing great answers.




                                          draft saved


                                          draft discarded














                                          StackExchange.ready(
                                          function () {
                                          StackExchange.openid.initPostLogin('.new-post-login', 'https%3a%2f%2fmath.stackexchange.com%2fquestions%2f3156341%2fwhy-is-xe-x0-for-all-x-in-mathbbr%23new-answer', 'question_page');
                                          }
                                          );

                                          Post as a guest















                                          Required, but never shown





















































                                          Required, but never shown














                                          Required, but never shown












                                          Required, but never shown







                                          Required, but never shown

































                                          Required, but never shown














                                          Required, but never shown












                                          Required, but never shown







                                          Required, but never shown







                                          Popular posts from this blog

                                          Magento 2 - Add success message with knockout Planned maintenance scheduled April 23, 2019 at 23:30 UTC (7:30pm US/Eastern) Announcing the arrival of Valued Associate #679: Cesar Manara Unicorn Meta Zoo #1: Why another podcast?Success / Error message on ajax request$.widget is not a function when loading a homepage after add custom jQuery on custom themeHow can bind jQuery to current document in Magento 2 When template load by ajaxRedirect page using plugin in Magento 2Magento 2 - Update quantity and totals of cart page without page reload?Magento 2: Quote data not loaded on knockout checkoutMagento 2 : I need to change add to cart success message after adding product into cart through pluginMagento 2.2.5 How to add additional products to cart from new checkout step?Magento 2 Add error/success message with knockoutCan't validate Post Code on checkout page

                                          Fil:Tokke komm.svg

                                          Where did Arya get these scars? Unicorn Meta Zoo #1: Why another podcast? Announcing the arrival of Valued Associate #679: Cesar Manara Favourite questions and answers from the 1st quarter of 2019Why did Arya refuse to end it?Has the pronunciation of Arya Stark's name changed?Has Arya forgiven people?Why did Arya Stark lose her vision?Why can Arya still use the faces?Has the Narrow Sea become narrower?Does Arya Stark know how to make poisons outside of the House of Black and White?Why did Nymeria leave Arya?Why did Arya not kill the Lannister soldiers she encountered in the Riverlands?What is the current canonical age of Sansa, Bran and Arya Stark?